summaryrefslogtreecommitdiffstats
diff options
context:
space:
mode:
authorJustin Gassner <justin.gassner@mailbox.org>2024-05-19 01:45:37 +0200
committerJustin Gassner <justin.gassner@mailbox.org>2024-05-19 01:45:37 +0200
commitb9e2609169709f8aad257fa5e3a92cb780dfad3f (patch)
tree380c67c4542013f6d9d8cf5e0ac658acd509c7bb
parentb3cf6a3ed2334719d5b7b047d5b5a6cbe4f14b30 (diff)
downloadmaster-b9e2609169709f8aad257fa5e3a92cb780dfad3f.tar.zst
weiter
-rw-r--r--.cspell.yaml39
-rw-r--r--analytic2.tex135
-rw-r--r--bib/articles.bib2
-rw-r--r--bib/misc.bib28
-rw-r--r--bib/much.bib29
-rw-r--r--bib/test.bib32
-rw-r--r--bib/test/distributions.bib170
-rw-r--r--main.tex4
-rw-r--r--much.tex502
-rw-r--r--preamble.tex37
-rw-r--r--stresstensor.tex302
11 files changed, 1205 insertions, 75 deletions
diff --git a/.cspell.yaml b/.cspell.yaml
index 8497152..16a83c3 100644
--- a/.cspell.yaml
+++ b/.cspell.yaml
@@ -1,38 +1,47 @@
----
-version: "0.2"
-language: "en-GB"
+version: 0.2
+language: en_US
+minWordLength: 3
dictionaryDefinitions:
- name: project-code
- path: "./.cspell/project-code.txt"
addWords: true
+ path: ./.cspell/project-code.txt
- name: project-words
- path: "./.cspell/project-words.txt"
addWords: true
+ path: ./.cspell/project-words.txt
- name: project-citekeys
- path: "./.cspell/project-citekeys.txt"
addWords: true
+ path: ./.cspell/project-citekeys.txt
- name: latex-pkgs
- path: "./.cspell/my-cspell-dicts/latex-pkgs.txt"
addWords: true
+ path: ./.cspell/my-cspell-dicts/latex-pkgs.txt
- name: biblatex
- path: "./.cspell/my-cspell-dicts/biblatex.txt"
addWords: true
+ path: ./.cspell/my-cspell-dicts/biblatex.txt
- name: names
- path: "./.cspell/my-cspell-dicts/names.txt"
addWords: true
+ path: ./.cspell/my-cspell-dicts/names.txt
+ - name: names2
+ addWords: true
+ path: ./.cspell/my-cspell-dicts/names2.txt
+ - name: math
+ addWords: true
+ path: ./.cspell/my-cspell-dicts/math.txt
dictionaries:
- latex
- project-code
- project-words
- latex-pkgs
+ - names
+ - names2
+ - math
+ignorePaths:
+ - ./.cspell/
+ - ./aux/
+ - ./cd-label/
+ - ./my/
+ - ./my2/
overrides:
- filename: "bib/**/*.bib"
dictionaries:
- biblatex
- names
-ignorePaths:
- - "./.cspell/"
- - "./aux/"
- - "./my/"
- - "./my2/"
-minWordLength: 3
diff --git a/analytic2.tex b/analytic2.tex
new file mode 100644
index 0000000..e0dc68f
--- /dev/null
+++ b/analytic2.tex
@@ -0,0 +1,135 @@
+\chapter{Analytic Vectors}
+
+\info{Dies ist nur ein Relikt meines Studiums analytischer Vektoren. Wird wieder entfernt, falls nicht benötigt.}
+
+\begin{definition}{Analytic Vector for an Operator}{analytic-vector-operator}
+ Let $A : D(A) \to \hilb{H}$ be an unbounded linear operator in a complex Hilbert space $\hilb{H}$.
+ A vector $x \in \hilb{H}$ is said to be an \emph{analytic vector} for $A$,
+ if $x$ lies in the domain of the power $A^n$ for all $n \in \NN$, and the power series
+ \begin{equation*}\tag{power-series-analytic-vector}
+ \sum_{n=0}^{\infty} \frac{A^n x}{n!} \, z^n
+ \end{equation*}
+ has a nonzero radius of convergece.
+ If the power series converges for all $z \in \CC$,
+ we say that $x$ is an \emph{entire analytic vector} for $A$.
+\end{definition}
+Note that, if $x$ is analytic for $A$, then the power series
+\begin{equation*}
+ \sum_{n=0}^{\infty} \frac{\norm{A^n x}}{n!} \, z^n
+\end{equation*}
+converges for all complex $z$ with $\abs{z} < t$,
+that is, in the open disc with radius $t$ centered in the origin of the complex plane.
+This is a well-known consequence of the convergence behavior of power series.
+
+\begin{definition}{Analyticity of Vector-Valued Functions}{}
+ Let $G \subset \CC$ be open and let $\hilb{H}$ be a Hilbert space.
+ A function $f : G \to \hilb{H}$ is called
+ \begin{itemize}
+ \item \emph{strongly analytic} at $a \in G$, if the limit
+ \begin{equation*}
+ \lim_{z \to a} \frac{f(z) - f(a)}{z-a}
+ \end{equation*}
+ exists in norm.
+ \item \emph{weakly analytic} in $a \in G$, if for each $w \in \CC$ the scalar-valued function
+ \begin{equation*}
+ G \longrightarrow \CC, \quad z \longmapsto \innerp{w}{f(z)}
+ \end{equation*}
+ is analytic in $a$.
+ \end{itemize}
+\end{definition}
+
+\begin{lemma}{Equivalence of Weak and Strong Analyticity}{}
+ Let $G \subset \CC$ be open.
+ Then a Banach space-valued function is strongly analytic on $G$ if and only if it is weakly analytic on $G$.
+\end{lemma}
+\begin{myproof}
+ Let $X$ be a Banach space and suppose that the function $f : G \to X$ is weakly analytic.
+ By definition, for each $g \in X'$ the scalar valued function $g \circ f : G \to \CC$ is analytic on $G$.
+ Consider a point $a \in G$.
+ Since $G$ is open, there exists a circular contour $\gamma$ around $a$ such that $\gamma$ and its interior lie wholly inside of $G$.
+ By Cauchy’s Integral Formula we have
+ \begin{equation*}
+ g(f(z)) = \frac{1}{2 \pi i} \int_{\gamma} \frac{g(f(w))}{w-z} \, dw
+ \end{equation*}
+ for any $z$ in the interior of $\gamma$.
+ Writing
+ \begin{equation*}
+ Q(z) = \frac{f(z) - f(a)}{z - a}
+ \end{equation*}
+ for the difference quotient, we get
+ \begin{equation*}
+ g(Q(z)) = \frac{1}{2 \pi i} \int_{\gamma} \frac{g(f(w))}{(w-z)(w-a)} \, dw
+ \end{equation*}
+ and
+ \begin{equation*}
+ g \parens*{\frac{Q(z) - Q(z')}{z - z'}} = \frac{1}{2 \pi i} \int_{\gamma} \frac{g(f(w))}{(w-z)(w-z')(w-a)} \, dw
+ \end{equation*}
+ for all $z,z'$ in the interior of $\gamma$.
+ The family of vectors $f(w) \in X$, indexed by complex numbers $w$ on the contour $\gamma$, can be viewed as a family of bounded linear functionals $C(f(w)) : X' \to \CC$
+ via the canonical embedding $C : X \to X''$ of $X$ into its bidual. For every fixed $g \in X'$ the set of values $C(f(w))(g) = g(f(w))$ is bounded, because the function $g \circ f$ is continous and the contour is compact.
+ In other words, the family of functionals $C(f(w))$, $w \in \gamma$, is pointwise bounded.
+ The Uniform Boundedness Theorem implies that there exists a constant $M > 0$ such that $\abs{g(f(w))} \le M \norm{g}$ for all $w$ on $\gamma$ and all $g \in X'$.
+ \begin{equation*}
+ \abs*{g \parens*{\frac{Q(z) - Q(z')}{z - z'}}} \le \frac{M}{2 \pi} \norm{g} \int_{\gamma} \frac{dw}{\abs{w-z}\abs{w-z'}\abs{w-a}}
+ \end{equation*}
+ If we restict $z,z'$ to a neighbourhood $N$ of $a$ that stays away from $\gamma$, then the integral on the right hand side is
+ bounded by a constant independent of $z$ and $z'$.
+ Absorbing all constants into $M' > 0$ we obtain
+ \begin{equation*}
+ \abs{g(Q(z) - Q(z'))} \le M' \norm{g} \abs{z-z'} \quad \forall z,z' \in N.
+ \end{equation*}
+ \begin{equation*}
+ \norm{Q(z) - Q(z')} = \sup_{\substack{g \in X'\\ \norm{g} \le 1}} \abs{g(Q(z) - Q(z'))} \le M' \norm{z - z'}.
+ \end{equation*}
+ Hence, the limit of $Q(z)$ for $z \to a$ exists by completeness of $X$.
+\end{myproof}
+
+\begin{definition}{Analytic Vector for an Unitary Group}{analytic-vector-unitary-group}
+ Let $\sigma : \RR \to U(\hilb{H})$ be a strongly continuous one-parameter unitary group on a complex Hilbert space $\hilb{H}$.
+ A vector $x \in \hilb{H}$ is said to be an \emph{analytic vector} for $\sigma$, if there exist
+ \begin{itemize}
+ \item a number $\lambda > 0$, defining a strip $I_{\lambda} = \braces{z : \abs{\Im z} < 1}$, and
+ \item a vector-valued function $f : I_{\lambda} \to \hilb{H}$,
+ \end{itemize}
+ with the properties that
+ \begin{itemize}
+ \item $f(t) = \sigma_t(x)$ for all $t \in \RR$,
+ \item $f$ is weakly analytic on $I_{\lambda}$.
+ \end{itemize}
+ In this case we write $f(z) = \sigma_z(x)$ for $z \in I_{\lambda}$.
+\end{definition}
+
+\begin{proposition}{}{}
+ Let $\sigma : \RR \to U(\hilb{H})$ be a strongly continuous one-parameter unitary group on a complex Hilbert space $\hilb{H}$
+ and let $A$ be its infinitesimal generator.
+ Then a vector $x \in \hilb{H}$ is analytic for $\sigma$ if and only if it is analytic for $A$.
+\end{proposition}
+
+\begin{myproof}
+ First, suppose that $x$ is an analytic vector for $\sigma$.
+ Then, there exist a number $\lambda > 0$ and a function $f : I_{\lambda} \to X$ as in \cref{definition:analytic-vector-unitary-group}.
+ In particular, $f$ is (strongly) analytic on the strip $I_{\lambda}$, which contains the disk $\braces{z : \abs{z} \le r}$ when $r \le \lambda$.
+ Hence we have Cauchy estimates
+ \begin{equation*}
+ \norm{f^{(n)}(0)} \le \frac{n!}{r^n} M \quad \forall n \in \NN,
+ \quad \text{where} \ M = \sup_{\abs{z} = r} \norm{f(z)}.
+ \end{equation*}
+ For real $t$ we have $f(t) = \sigma_t(x) = \exp(itA) x$ and
+ the mapping $t \mapsto f(t)$ is strongly differentiable with derivatives
+ $f^{(n)}(0) = (iA)^n x$.
+ This implies that the power series
+ \begin{equation*}
+ \sum_{n=0}^{\infty} \frac{\norm{A^n x}}{n!} t^n \le M \sum_{n=0}^{\infty} \frac{t^n}{r^n}
+ \end{equation*}
+ is convergent for $t \le \lambda$ by majorization. Hernce $x$ is an analytic vector for the operator $A$.
+
+ Coversely, suppose that $x$ is analytic for the generator $A$ of $\sigma$.
+ Then, by \cref{definition:analytic-vector-operator}, $x$ lies in the domains of all powers $A^n$, $n \in \NN$, and the power series
+ \begin{equation*}
+ \sum_{n=0}^{\infty} \frac{\norm{A^n x}}{n!} z^n
+ \end{equation*}
+ has a positive radius of convergence $t>0$.
+\end{myproof}
+
+\chapterbib
+\cleardoublepage
diff --git a/bib/articles.bib b/bib/articles.bib
index 2368b40..60d4d4f 100644
--- a/bib/articles.bib
+++ b/bib/articles.bib
@@ -1,8 +1,8 @@
@article{Epstein1965,
author = {Epstein, H. and Glaser, V. and Jaffe, A.},
title = {Nonpositivity of the Energy Density in Quantized Field Theories},
- publisher = {Società Italiana di Fisica},
journaltitle = {Il Nuovo Cimento},
+ publisher = {Società Italiana di Fisica},
volume = {36},
issue = {3},
date = {1965},
diff --git a/bib/misc.bib b/bib/misc.bib
new file mode 100644
index 0000000..b32be08
--- /dev/null
+++ b/bib/misc.bib
@@ -0,0 +1,28 @@
+@book{ReedSimon1,
+ maintitle = {Methods of Modern Mathematical Physics},
+ title = {Functional Analysis},
+ author = {Michael Reed and Barry Simon},
+ publisher = {Academic Press},
+ date = {1980},
+ edition = {Revised and Enlarged Edition},
+ volume = {1},
+}
+@book{ReedSimon2,
+ maintitle = {Methods of Modern Mathematical Physics},
+ title = {Fourier Analysis, Self-Adjointness},
+ author = {Michael Reed and Barry Simon},
+ publisher = {Academic Press},
+ date = {1975},
+ volume = {2},
+}
+@article{Nelson1972,
+ title = {Time-Ordered Operator Products of Sharp-Time Quadratic Forms},
+ author = {Edward Nelson},
+ publisher = {Elsevier Science},
+ journal = {Journal of Functional Analysis},
+ issn = {0022-1236,1096-0783},
+ date = {1972},
+ volume = {11},
+ number = {2},
+ pages = {211--219},
+}
diff --git a/bib/much.bib b/bib/much.bib
new file mode 100644
index 0000000..c2f76f4
--- /dev/null
+++ b/bib/much.bib
@@ -0,0 +1,29 @@
+@misc{Much2022,
+ title={An approximate local modular quantum energy inequality in general quantum field theory},
+ author={Albert Much and Albert Georg Passegger and Rainer Verch},
+ year={2022},
+ eprint={2210.01145},
+ archivePrefix={arXiv},
+ primaryClass={math-ph}
+}
+@article{Bisognano1975,
+ title = {On the duality condition for a Hermitian scalar field},
+ author = {Joseph J. Bisognano and Eyvind H. Wichmann},
+ publisher = {American Institute of Physics},
+ journal = {Journal of Mathematical Physics},
+ issn = {0022-2488,1089-7658},
+ year = {1975},
+ volume = {16},
+ number = {4},
+ pages = {985-1007},
+}
+@book{Schmüdgen2012,
+ title = {Unbounded Self-adjoint Operators on Hilbert Space},
+ author = {Konrad Schmüdgen},
+ publisher = {Springer},
+ isbn = {9789400747524},
+ year = {2012},
+ series = {Graduate Texts in Mathematics},
+ edition = {1},
+ volume = {265},
+}
diff --git a/bib/test.bib b/bib/test.bib
new file mode 100644
index 0000000..c6d07d6
--- /dev/null
+++ b/bib/test.bib
@@ -0,0 +1,32 @@
+@book{book:4004309,
+ title = {Introduction to Algebraic and Constructive Quantum Field Theory},
+ author = {John C. Baez and Irving Ezra Segal and Zhengfang Zhou},
+ publisher = {Princeton University Press},
+ isbn = {9781400862504},
+ year = {1992},
+ series = {Princeton Series in Physics},
+ volume = {},
+}
+@article{Wick1950,
+ title = {The Evaluation of the Collision Matrix},
+ author = {G. C. Wick},
+ publisher = {American Physical Society},
+ journal = {Physical Review},
+ issn = {0031-899X,1536-6065},
+ year = {1950},
+ volume = {80},
+ number = {2},
+ pages = {268--272},
+}
+@article{Fewster1998,
+ title = {Bounds on negative energy densities in flat spacetime},
+ author = {C. J .Fewster and S. P. Eveson},
+ publisher = {American Physical Society},
+ journal = {Physical Review D},
+ ISSN = {1089-4918},
+ year = {1998},
+ volume = {58},
+ number = {8},
+ month = sep,
+ pages = {084010},
+}
diff --git a/bib/test/distributions.bib b/bib/test/distributions.bib
new file mode 100644
index 0000000..81746dc
--- /dev/null
+++ b/bib/test/distributions.bib
@@ -0,0 +1,170 @@
+@book{Halperin1952,
+ series = {Canadian Mathematical Congress, Lecture Series},
+ volume = {1},
+ title = {Introduction to the Theory of Distributions},
+ author = {Israel Halperin},
+ note = {Based on the lectures given by \textsc{L. Schwartz}.},
+ publisher = {University of Toronto Press},
+ year = {1952},
+}
+@book{Lighthill1958,
+ title = {Introduction to Fourier Analysis and Generalised Functions},
+ author = {M. J. Lighthill},
+ publisher = {Cambridge University Press},
+ year = {1958},
+}
+@article{Gårding1959,
+ doi = {10.1007/bf02724838},
+ title = {Functional Analysis},
+ author = {L. Gårding and J. L. Lions},
+ publisher = {Società Italiana di Fisica},
+ journaltitle = {Il Nuovo Cimento},
+ issn = {0029-6341,1827-6121},
+ year = {1959},
+ volume = {14},
+ issue = {1 Supplement},
+ pages = {9--66},
+}
+@book{Erdélyi1962,
+ title = {Operational Calculus and Generalized Functions},
+ author = {Arthur Erdélyi},
+ publisher = {Holt, Rinehart and Winston},
+ year = {1962},
+}
+@book{Schwartz1966,
+ title = {Théorie des distributions},
+ author = {Laurent Schwartz},
+ publisher = {Hermann},
+ year = {1966},
+ language = {french},
+}
+@book{Trèves1967,
+ title = {Topological Vector Spaces, Distributions and Kernels},
+ author = {François Trèves},
+ publisher = {Academic Press},
+ year = {1967},
+}
+@book{Jones1982,
+ title = {The Theory of Generalised Functions},
+ author = {D. S. Jones},
+ publisher = {Cambridge University Press},
+ year = {1982},
+ edition = {2},
+}
+@book{GenFunc1,
+ maintitle = {Generalized Functions},
+ volume = {1},
+ title = {Properties and Operations},
+ author = {I. M. Gel'fand and G. E. Shilov},
+ publisher = {Academic Press},
+ year = {1964},
+ language = {english},
+ origlanguage = {russian},
+ translator = {Eugene Saletan},
+}
+@book{GenFunc2,
+ maintitle = {Generalized Functions},
+ volume = {2},
+ title = {Spaces of Fundamental and Generalized Functions},
+ author = {I. M. Gel'fand and G. E. Shilov},
+ publisher = {Academic Press},
+ year = {1968},
+ language = {english},
+ origlanguage = {russian},
+ translator = {Morris D. Friedman and Amiel Feinstein and Christian P. Peltzer},
+}
+@book{GenFunc3,
+ maintitle = {Generalized Functions},
+ volume = {3},
+ title = {Theory of Differential Equations },
+ author = {I. M. Gel'fand and G. E. Shilov},
+ publisher = {Academic Press},
+ year = {1967},
+ language = {english},
+ origlanguage = {russian},
+ translator = {Meinhard E. Mayer},
+}
+@book{GenFunc4,
+ maintitle = {Generalized Functions},
+ volume = {4},
+ title = {Applications of Harmonic Analysis},
+ author = {I. M. Gel'fand and N. Ya. Vilenkin},
+ publisher = {Academic Press},
+ year = {1964},
+ language = {english},
+ origlanguage = {russian},
+ translator = {Amiel Feinstein},
+}
+@book{GenFunc5,
+ maintitle = {Generalized Functions},
+ volume = {5},
+ title = {Integral Geometry and Representation Theory},
+ author = {I. M. Gel'fand and M. I. Graev and N. Ya. Vilenkin},
+ publisher = {Academic Press},
+ year = {1966},
+ language = {english},
+ origlanguage = {russian},
+ translator = {Eugene Saletan},
+}
+@book{GenFunc6,
+ maintitle = {Generalized Functions},
+ volume = {6},
+ title = {Representation Theory and Automorphic Functions},
+ author = {I. M. Gel'fand and M. I. Graev and I. I. Pyatetskii-Shapiro},
+ publisher = {W. B. Saunders},
+ year = {1969},
+ language = {english},
+ origlanguage = {russian},
+ translator = {K. A. Hirsch},
+}
+@book{Richards1990,
+ title = {The Theory of Distributions},
+ subtitle = {A Nontechnical Introduction},
+ author = {Ian Richards and Heekyung Youn},
+ publisher = {Cambridge University Press},
+ year = {1990},
+}
+@book{Strichartz1994,
+ title = {A Guide to Distribution Theory and Fourier Transforms},
+ author = {Robert Strichartz},
+ publisher = {CRC Press},
+ year = {1994},
+ series = {Studies in Advanced Mathematics},
+ edition = {},
+ volume = {},
+}
+@article{Wightman1996,
+ doi = {10.1002/prop.2190440204},
+ title = {How It Was Learned that Quantized Fields Are Operator-Valued Distributions},
+ author = {A. S. Wightman},
+ publisher = {John Wiley and Sons},
+ journaltitle = {Fortschritte der Physik},
+ issn = {0015-8208,1521-3978},
+ year = {1996},
+ volume = {44},
+ issue = {2},
+ pages = {143--178},
+}
+@book{Friedlander1999,
+ title = {Introduction to the Theory of Distributions},
+ author = {F. G. Friedlander and M. Joshi},
+ publisher = {Cambridge University Press},
+ year = {1999},
+ edition = {2},
+}
+@book{Hoskins2005,
+ title = {Theories of Generalised Functions},
+ title = {Distributions, Ultradistributions and Other Generalised Functions},
+ author = {R. F. Hoskins and J. S. Pinto},
+ publisher = {Woodhead Publishing},
+ year = {2005},
+ edition = {2},
+}
+@book{Hoskins2009,
+ title = {Delta Functions},
+ subtitle = {Introduction to Generalised Functions},
+ author = {R. F. Hoskins},
+ publisher = {Woodhead Publishing},
+ year = {2009},
+ edition = {2},
+}
diff --git a/main.tex b/main.tex
index fc44017..91aaaab 100644
--- a/main.tex
+++ b/main.tex
@@ -1,5 +1,5 @@
\input{preamble}
-\includeonly{stresstensor,fewstereveson,commutatortheorem,index}
+\includeonly{stresstensor,fewstereveson,much,commutatortheorem,analytic2,symbols,index}
\begin{document}
\frontmatter
\include{titlepage}
@@ -12,10 +12,12 @@
\include{standard}
\include{stresstensor}
\include{fewstereveson}
+\include{much}
\include{samplesection}
\appendix
\include{sampleappendix}
\include{commutatortheorem}
+\include{analytic2}
\backmatter
\include{bibliography}
\include{symbols}
diff --git a/much.tex b/much.tex
new file mode 100644
index 0000000..58aeab3
--- /dev/null
+++ b/much.tex
@@ -0,0 +1,502 @@
+\chapter{A quantum energy inequality involving local modular data}
+
+
+\cite{Much2022}
+
+\begin{equation*}
+ \innerp{\psi}{\energydensity(f)\psi} \ge
+ - \epsilon - \norm{\smash[b]{\Delta}_{\smash[t]{\sharp}}^{-1/2} \ft{g}_{\lambda}(K_{\raisebox{5pt}{\footnotesize$\sharp$}}) \energydensity(f) \fockvaccum}
+\end{equation*}
+
+
+\section{Misc}
+
+\todo{Put this somwhere else.}
+
+A \emph{Lorentz transform} is a linear automorphism of Minkowski spacetime
+which preserves the Lorentz bilinear form.
+Lorentz transforms are usually represented by (real) $4 \times 4$ matrices,
+with respect to the standard basis.
+the \emph{Lorentz group} $\FullLorentzGroup$.
+\begin{equation*}
+ \FullPoincareGroup = \RR^4 \ltimes \FullLorentzGroup
+\end{equation*}
+
+The relativistic transformation law for one-particle states is given by
+\begin{equation*}
+ \parens[\big]{U(a,\Lambda) \psi}(p) = e^{ia \cdot p} \psi(\Lambda^{-1} p),
+ \quad \psi \in \hilb{H}, (a,\Lambda) \in \ProperOrthochronousPoincareGroup.
+\end{equation*}
+The mapping $(a,\Lambda) \mapsto U(a,\Lambda)$ is a (irreducible) unitary representation
+of the proper orthochronous Poincaré group $\ProperOrthochronousPoincareGroup$
+on the one-particle Hilbert space.
+By applying any of the second quantization functors we obtain a representation on the multi-particle state space.
+\begin{equation*}
+ \parens[\big]{U(a,\Lambda) \psi}{}_n(p_1,\ldots,p_n) = e^{ia \cdot (p_1 + \cdots + p_n)} \psi_n(\Lambda^{-1} p_1, \ldots, \Lambda^{-1} p_n),
+\end{equation*}
+
+Poincaré covariance
+\begin{equation}
+ \label{equation:poincare-covariance-local-algebras}
+ U(g) \localalg{\spacetimeregion{O}} U(g)^* = \localalg{g\spacetimeregion{O}}
+ \qquad g \in \ProperOrthochronousPoincareGroup
+\end{equation}
+
+\begin{definition}{Von Neumann Algebra of Local Observables}{}
+ \begin{equation*}
+ \localalg{\spacetimeregion{O}} = \braces{b(\varphi(f)) \mid b, f \in \realschwartz{M}, \supp f \subset \spacetimeregion{O}}''
+ \end{equation*}
+\end{definition}
+
+\section{Basic Concepts of Modular Theory}
+\index{modular!theory}
+
+If $\hilb{H}$ is a Hilbert space
+we shall denote the $C^*$-algebra of all bounded linear operators on $\hilb{H}$ by $B(\hilb{H})$.
+
+\begin{definition}{Cyclic and Separating Vectors}{}
+ Suppose $\hilb{H}$ is a Hilbert space and $\mathcal{A}$ is a $C^*$-subalgebra of $B(\hilb{H})$.
+ A vector $\Omega \in \hilb{H}$ is called
+ \begin{itemize}
+ \item \emph{cyclic}\index{cyclic vector} for $\mathcal{A}$ if the vector set $\mathcal{A} \Omega$ is dense in $\hilb{H}$.
+ \item \emph{separating}\index{separating vector} for $\mathcal{A}$ if the map $A \mapsto A \Omega$ from $\mathcal{A}$ into $\hilb{H}$ is injective.
+ \end{itemize}
+\end{definition}
+Occasionally, a vector that is both cyclic and separating is called \emph{standard}\index{standard vector}.
+
+Recall that the commutant of a set $\mathcal{S} \subset B(\hilb{H})$ of operators
+is defined as the set of all operators $T \in B(\hilb{H})$ which commute with all operators $S$ in $\mathcal{S}$.
+We shall denote the commutant of $\mathcal{S}$ by $\mathcal{S}'$.\nomenclature{$\mathcal{A}'$}{commutant of $\mathcal{A}$}
+
+\begin{proposition}{}{cyclic-separating}
+ \begin{enumerate}[label=(\roman*),nosep,leftmargin=*,widest=ii]
+ \item A vector is cyclic for $\mathcal{A}$ if and only if it is separating for $\mathcal{A}'$.
+ \item If $\vNa{M}$ is a von Neumann algebra, then a vector is cyclic and separating for $\vNa{M}$
+ if and only if it is cyclic and separating for $\vNa{M}'$.
+ \end{enumerate}
+\end{proposition}
+
+\begin{proof}
+ \todo{xxx}
+ The second assertion directly follows from the first and the fact that $\vNa{M}'' = \vNa{M}$.
+\end{proof}
+
+If $\Omega$ is separating for $\mathcal{A}$,
+then every element of $\mathcal{A}\Omega$ is of the form $A\Omega$
+with a unique $A \in \mathcal{A}$.
+This allows us to define an (anti-linear) operator $S_0$ in $\hilb{H}$ with domain $\mathcal{A}\Omega$ by
+\begin{equation}
+ \label{equation:definition-s0}
+ \quad S_0 A\Omega \defequal S_0 A^*\Omega \qquad A \in \mathcal{A}.
+\end{equation}
+The operator $S_0$ is densely defined if and only if $\Omega$ is cyclic for $\mathcal{A}$.
+Since the $*$-operation on $\mathcal{A}$ is involutive,
+the range of $S_0$ coincides with its domain.
+
+\begin{lemma}{}{}
+ If $\Omega$ is a cyclic and separating vector for a von Neumann algebra $\mathcal{A}$,
+ then the operator $S_0$ defined by~\eqref{equation:definition-s0} is closable.
+\end{lemma}
+\begin{proof}
+ By \cref{proposition:cyclic-separating},
+ $\Omega$ is also cyclic and separating for the commutant $\vNa{A}'$.
+ Hence we may, analogously to $S_0$,
+ define another anti-linear operator $F_0$ in $\hilb{H}$ with dense domain $\mathcal{A}' \Omega$ by
+ \begin{equation*}
+ \quad F_0 B\Omega \defequal F_0 B^*\Omega \qquad B \in \mathcal{A'}.
+ \end{equation*}
+ By definition of $S_0$ and $F_0$ we have for every $A \in \mathcal{A}$ and $B \in \mathcal{A}'$
+ \begin{equation*}
+ \innerp{S_0 A \Omega}{B \Omega} =
+ \innerp{\Omega}{AB \Omega} =
+ \innerp{\Omega}{BA \Omega} =
+ \innerp{F_0 B\Omega}{A \Omega}.
+ \end{equation*}
+ This adjoint identity establishes that $S_0 \subset F_0^*$.
+ (The \enquote{twisted} appearance of the identity is correct,
+ since it involves anti-linear operators on both sides.)
+ The Hilbert adjoint $F_0^*$ of $F_0$ is closed.
+ Hence, we have shown that $S_0$ has a closed extension, and
+ this implies that $S_0$ is closable.
+\end{proof}
+
+\begin{definition}{Tomita operator}{}
+ Suppose $\Omega$ is a cyclic and separating vector for a von Neumann algebra $\mathcal{A}$.
+ The closure $S = \operatorclosure{S_0}$
+ of the operator $S_0$ defined on $\mathcal{A}\Omega$ by
+ $S_0 A\Omega = S_0 A^*\Omega$
+ for $A \in \mathcal{A}$
+ is called the
+ \emph{Tomita operator}\index{Tomita operator}\index{operator!Tomita}\nomenclature{$S$}{Tomita operator}
+ for the pair $(\mathcal{A},\Omega)$.
+\end{definition}
+
+It is a well-known fact that closed operators can be decomposed
+in a similar fashion to the polar coordinate representation $z = e^{i\arg z} \abs{z}$
+of a complex number.
+We state the theorem in its somewhat uncommon variant for anti-linear operators,
+as this is our only use case.
+
+\begin{theorem}{Polar Decomposition for Anti-Linear Closed Operators}{polar-decomposition}
+ \index{polar decomposition}
+ Let $T$ be an arbitrary closed anti-linear operator in a Hilbert space $\hilb{H}$.
+ Then there exist
+ a positive selfadjoint linear operator $\abs{T}$ and
+ a partial anti-linear isometry $U$
+ such that
+ \begin{equation*}
+ T = U \abs{T} \qquad \bracks[\big]{\text{in particular, $\Domain{T} = \Domain{\abs{T}}$}}.
+ \end{equation*}
+ The operators $U$ and $\abs{T}$ are uniquely determined given the additional conditions
+ \begin{equation*}
+ \ker\abs{T} = \ker T \qquad
+ (\ker U)^\perp = (\ker T)^\perp \qquad
+ \ran U = \overline{\ran T}.
+ \end{equation*}
+\end{theorem}
+
+Proofs of this statement are contained in~\cite{ReedSimon1} and~\cite{Schmüdgen2012}.
+When we speak of \emph{the} polar composition we tacitly assume that the additional conditions
+ensuring uniqueness are satisfied.
+
+Now we are able to introduce the fundamental objects of modular theory.
+
+\begin{definition}{Modular Conjugation, Modular Operator}{}
+ Suppose $\vNa{M}$ is a von Neumann algebra acting on a Hilbert space $\hilb{H}$,
+ and suppose $\Omega \in \hilb{H}$ is a cyclic and separating vector for $\vNa{M}$.
+ Let $S$ be the Tomita operator for $(\vNa{M},\Omega)$ and let
+ \begin{equation*}
+ S = J \Delta^{1/2}
+ \end{equation*}
+ be its polar decomposition.
+ The anti-unitary operator $J$ is called
+ \emph{modular conjugation}\index{modular!conjugation}\nomenclature{$J$}{modular conjugation}.
+ The positive selfadjoint operator $\Delta$ is called
+ \emph{modular operator}\index{modular!operator}\index{operator!modular}\nomenclature{$\Delta$}{modular operator}.
+ The pair $(J,\Delta)$ is said to be the \emph{modular data}\index{modular!data}\index{modular!objects} associated to
+ the pair $(\vNa{M},\Omega)$.
+\end{definition}
+
+\todo{clarify why $J$ is anti-unitary}
+
+\begin{definition}{Modular Group}{}
+ Adopt the notation of the foregoing definition.
+ The mapping $\RR \ni t \mapsto \Delta^{it}$ is called the \emph{modular group}\index{modular!group} associated to
+ $(\vNa{M},\Omega)$.
+\end{definition}
+
+The modular group is a strongly continuous one-parameter unitary group on $\hilb{H}$.
+
+\newpage
+
+\begin{proposition}{}{modular-data-unitary}
+ Suppose $\vNa{M}$ is a von Neumann algebra acting on a Hilbert space $\hilb{H}$.
+ Let $U$ be a unitary operator on $\hilb{H}$.
+ Then $U\vNa{M}U^*$ is a von Neumann algebra on $\hilb{H}$.
+ Suppose further that $\Omega \in \hilb{H}$ is a cyclic and separating vector for $\vNa{M}$.
+ Then $U \Omega$ is cyclic and separating for $U\vNa{M}U^*$.
+ Let $(J,\Delta)$ be the modular data associated to $(\vNa{M},\Omega)$.
+ Then $(UJU^*,U{\Delta}U^*)$ is the modular data associated to $(U\vNa{M}U^*,U\Omega)$.
+\end{proposition}
+
+\begin{proof}
+ To prove the first assertion,
+ consider any $A \in (U\vNa{M}U^*)''$.
+ By the double commutant theorem,
+ it suffices to show that $A \in U\vNa{M}U^*$.
+ As $\vNa{M}$ is a von Neumann algebra,
+ this is equivalent to $U^*\! AU \in \vNa{M}''$,
+ again by the double commutant theorem.
+ Let $B \in \vNa{M}'$.
+ It is easy to check that $UBU^* \in (U\vNa{M}U^*)'$.
+ By assumption, $A$ lies in the commutant of $(U\vNa{M}U^*)'$.
+ Thus we find that $[U^*\! AU,B] = U^* [A,UBU^*] U = 0$, as desired.
+
+ The set of vectors $U\vNa{M}U^* U\Omega = U\vNa{M}\Omega$ is dense in $\hilb{H}$,
+ since it is the image of $\vNa{M} \Omega$ under the homeomorphism $U$.
+ Thus, the vector $U\Omega$ is cyclic for $U\vNa{M}U^*$.
+ Let us show that it is also separating.
+ Suppose $A$ is in $\vNa{M}$ and $UAU^*U\Omega = UA\Omega = 0$
+ Since unitaries are injective, $A\Omega = 0$.
+ Now $A=0$ follows from the assumption that $\Omega$ is separating for $\vNa{M}$.
+ We have shown that the mapping $UAU^*U\Omega = UA\Omega$ from $U\vNa{M}U^* \to \hilb{H}$ is injective.
+
+ Let $S = \overline{S_0}$ be the Tomita operator associated to $(\vNa{M},\Omega)$,
+ and let $S' = \overline{S'_0}$ be the Tomita operator associated to $(U\vNa{M}U^*,U\Omega)$.
+ Then we have
+ \begin{equation*}
+ (S'_0 U) A \Omega =
+ S'_0 (U A U^*) U \Omega =
+ (U A^* U^*) U \Omega =
+ U A^* \Omega =
+ U S_0 A \Omega
+ \end{equation*}
+ for all $A \in \vNa{M}$. Consequently, $S'_0 = U S_0 U^*$ as operators with domain $U\vNa{M}\Omega$.
+ Taking the closure, we obtain $S' = U S U^*$.
+ We can write this as $S' = UJU^* U\Delta^{1/2} U^*$,
+ where $S = J \Delta^{1/2}$ is the polar decomposition of the Tomita operator.
+ It is straightforward to check that
+ $UJU^*$ is anti-unitary and $U\Delta^{1/2} U^*$ is positive selfadjoint,
+ and satisfy the additional condition of \cref{theorem:polar-decomposition}.
+ The uniqueness of the polar decomposition implies that
+ $UJU^*$ is the modular conjugation and $U\Delta U^*$ is the modular conjugation
+ associated to the pair $(U\vNa{M}U^*,U\Omega)$.
+\end{proof}
+
+\newpage
+
+Finally, let us outline how modular theory enters into algebraic quantum field theory.
+
+\begin{theorem}{Reeh-Schlieder Theorem}{reeh-schlieder}
+ \todo{spell it out}
+\end{theorem}
+
+By Reeh-Schlieder (\cref{theorem:reeh-schlieder}), the vacuum $\Omega$ is cyclic and separating for $\localalg{\spacetimeregion{O}}$.
+Thus, modular theory
+
+
+\section{The Geometric Action of the Modular Operator Associated With a Wedge Domain}
+
+\begin{definition}{Right and Left Wedge, General Wedges}{}
+ The \emph{right wedge}\index{wedge!right}\nomenclature[WR]{$\rightwedge$}{right wedge}
+ and \emph{left wedge}\index{wedge!left}\nomenclature[WL]{$\leftwedge$}{left wedge}
+ in Minkowski space $M$ are the open subsets
+ \begin{equation*}
+ \rightwedge \defequal \braces[\big]{x \in M \vcentcolon x^1 > \abs{x^0}}
+ \quad \text{and} \quad
+ \leftwedge \defequal \braces[\big]{x \in M \vcentcolon x^1 < -\abs{x^0}}.
+ \end{equation*}
+We say that a spacetime region $W \subset M$ is a \emph{wedge}\index{wedge}
+ if there exists an element $g$ of the Poincaré group
+ such that $W = g \rightwedge$.
+\end{definition}
+
+Instead of the right wedge,
+we could just as well have used the left wedge to define the notion of a general wedge,
+since they are transformed into each other by space inversion.
+
+\begin{lemma}{}{general-wedge-from-right-wedge}
+ If a spacetime region $W$ is a wedge,
+ then there exists an element $g$ of the proper orthochronous Poincaré group
+ such that $W = g \rightwedge$.
+\end{lemma}
+
+\begin{proof}
+ \todo{xxx}
+\end{proof}
+
+In the standard representation of the Lorentz group, the boost (or velocity transformation) along the $x^1$-axis
+with rapidity $2 \pi t$ is given by the matrix\footnote{
+ This matrix depends on the choice of metric signature.
+ Ours is $(+,-,-,-)$.
+ For $(-,+,+,+)$, use
+ \begin{equation*}
+ \Lambda(t) = \begin{pmatrix}
+ \phantom{-}\cosh(2 \pi @ t) & -\sinh(2 \pi @ t) & \; 0 \; & \; 0 \; \\
+ -\sinh(2 \pi @ t) & \phantom{-}\cosh(2 \pi @ t) & 0 & 0 \\
+ 0 & 0 & 1 & 0 \\
+ 0 & 0 & 0 & 1 \\
+ \end{pmatrix}.
+ \end{equation*}
+ }
+
+\begin{equation*}
+ \Lambda(t) = \begin{pmatrix}
+ \cosh(2 \pi @ t) & \sinh(2 \pi @ t) & \; 0 \; & \; 0 \; \\
+ \sinh(2 \pi @ t) & \cosh(2 \pi @ t) & 0 & 0 \\
+ 0 & 0 & 1 & 0 \\
+ 0 & 0 & 0 & 1 \\
+ \end{pmatrix}
+\end{equation*}
+
+The following proposition shows that $t \mapsto \Lambda(t)$ is
+a one-parameter subgroup of the stabilizer group of the right wedge
+with respect to the action of the Lorentz group on subsets of Minkowski space.
+
+\begin{proposition}{}{}
+ \begin{enumerate}[label=(\roman*),nosep,leftmargin=*,widest=ii]
+ \item $\Lambda(s + t) = \Lambda(s) \Lambda(t)$ for all $s,t \in \RR$.
+ \item $\Lambda(t) \rightwedge = \rightwedge$ for all $t \in \RR$.
+ \end{enumerate}
+\end{proposition}
+
+\begin{proof}
+ The first property can be verified by direct computation.
+ Let us prove the second.
+ By definition, the image $\Lambda(t) x$ of a vector $x \in M$ lies in $\rightwedge$ if and only if
+ \begin{equation*}
+ x^0 \sinh(2 \pi t) + x^1 \cosh(2 \pi t)
+ > \abs{x^0 \cosh(2 \pi t) + x^1 \sinh(2 \pi t)},
+ \end{equation*}
+ or equivalently
+ \begin{equation*}
+ x^0 \parens[\big]{\sinh(2 \pi t) \mp \cosh(2 \pi t)}
+ + x^1 \parens[\big]{\cosh(2 \pi t) \mp \sinh(2 \pi t)} > 0
+ \end{equation*}
+ for both sign choices.
+ Using the definitions of the hyperbolic sine and cosine, this may be further simplified to $(x^1 \mp x^0) e^{2 \pi t} > 0$,
+ which holds if and only if $x^1 > \abs{x^0}$,
+ since the exponential is always positive.
+ So we have shown that
+ \begin{equation}
+ \label{equation:image-right-wedge}
+ \Lambda(t) x \in \rightwedge \iff x \in \rightwedge.
+ \end{equation}
+ This implies that $\Lambda(t)\rightwedge \subset \rightwedge$ for all $t \in \RR$.
+ Conversely, given an arbitrary vector $y \in \rightwedge$,
+ we have to find $x \in \rightwedge$ such that $\Lambda(t) x = y$.
+ Consider $x = \Lambda(-t) y$. Clearly, $x \in \rightwedge$, because of $y \in \rightwedge$
+ and~\eqref{equation:image-right-wedge}. Now it follows from $\Lambda(-t) = \Lambda(t)^{-1}$ that in fact $\Lambda(t) x = y$.
+\end{proof}
+
+\begin{theorem}{Bisognano-Wichmann Theorem \textmd{\cite{Bisognano1975}}}{}
+ For the theory of a free scalar field in Minkowski spacetime,
+ let $\spacetimeregion{O} \mapsto \localalg{\spacetimeregion{O}}$ be the net of von Neumann algebras of local observables.
+ If $(J,\Delta)$ is the modular data associated to the algebra $\localalg{\rightwedge}$ of the right wedge and the vacuum $\Omega$, then
+ \begin{equation*}
+ J = \Theta \cdot U\parens[\big]{0, R_{23}(\pi)} \qquad
+ \Delta^{it} = U\parens[\big]{0,\Lambda(t)},
+ \end{equation*}
+ where $U$ is the theory's unitary representation of the proper orthochronous Poincaré group.
+\end{theorem}
+
+\todo{give definition of $\Theta$ and $R_{23}$}
+
+Note that above statement is for the right wedge only.
+Let us investigate how the modular group changes, if we consider another wedge region.
+By \cref{lemma:general-wedge-from-right-wedge} any wedge $W$ can be obtained as $W = g\rightwedge$, where $g$ is a proper orthochronous Poincaré transformation.
+The covariance property~\eqref{equation:poincare-covariance-local-algebras} of $\vNa{R}$ implies
+\begin{equation*}
+ \localalg{W} =
+ U(g) \localalg{\rightwedge} U(g)^*.
+\end{equation*}
+The vacuum $\Omega$ is Poincaré invariant:
+\begin{equation*}
+ U(g) \Omega = \Omega.
+\end{equation*}
+We write $(J_W,\Delta_W)$ for the modular data associated to $(\localalg{W},\Omega)$.
+By \cref{proposition:modular-data-unitary}
+\begin{equation*}
+ J_W = U(g) J U(g)^* \qquad
+ \Delta_W = U(g) \Delta U(g)^*
+\end{equation*}
+Recall that the modular group $\Delta_W^{it}$ is defined by means of functional calculus.
+This raises the following problem: given a selfadjoint operator $A$, a unitary operator $U$
+and a suitable function $f$ we want to express $f(UAU^*)$ in terms of $f(A)$, if possible.
+Note that two different functional calculi are at play here, the former is for $UAU^*$ and the latter for $A$.
+Simple functions such as polynomials suggest $f(UAU^*) = Uf(A)U^*$.
+That this is generally true is the statement of the following Lemma.
+
+
+\begin{lemma}{}{functional-calclus-unitary-trafo}
+ Suppose that $A$ is a selfadjoint operator on a Hilbert space $\hilb{H}$,
+ with spectral measure $E_A$.
+ Suppose $U$ is an unitary operator on $\hilb{H}$, and
+ let $E_{U\! @AU^*}$ denote the spectral measure of the (selfadjoint) operator $UAU^*$.
+ Then we have $U E_A U^* = E_{U\! @AU^*}$, and
+ \begin{equation*}
+ U f(A) U^* = f(U\! @@AU^*)
+ \end{equation*}
+ for all Borel functions $f : \RR \to \CC$.
+\end{lemma}
+
+\question{Ist diese Aussage korrekt? Ist mein Beweis richtig? Geht der auch einfacher?}
+
+\begin{proof}
+ For each regular value $\lambda \in \rho(A)$ let
+ \begin{equation*}
+ R_A(\lambda) = (A-\lambda)^{-1}
+ \end{equation*}
+ denote the resolvent operator of $A$.
+ This proof is based on Stone's Formula \todo{reference},
+ which relates the resolvent to the spectral projections of $A$:
+ If $E_A$ is the spectral measure of $A$ and $\alpha < \beta$ are real numbers, then
+ \begin{equation*}
+ \stronglim_{\varepsilon \downarrow 0}
+ \frac{1}{\pi i} \int_{\alpha}^{\beta} \bracks{R_A(\lambda + i \varepsilon) - R_A(\lambda - i \varepsilon)} d\lambda
+ = E_A \parens[\big]{\bracks{\alpha,\beta}} + E_A \parens[\big]{\parens{\alpha,\beta}}
+ \end{equation*}
+ Recall that a spectral measure is countably additive.
+ As a consequence,
+ \begin{equation*}
+ \stronglim_{\alpha \uparrow a}
+ \stronglim_{\beta \downarrow b}
+ \stronglim_{\varepsilon \downarrow 0}
+ \frac{1}{2\pi i} \int_{\alpha}^{\beta} \bracks{R_A(\lambda + i \varepsilon) - R_A(\lambda - i \varepsilon)} d\lambda
+ = E_A \parens[\big]{\bracks{a,b}}
+ \end{equation*}
+ for all $a \in \RR \cup \braces{-\infty}$, $b \in \RR \cup \braces{\infty}$.
+ Observe that $\rho(A) = \rho(U\! @AU^*)$ and that for each (common) regular value $\lambda$ we have
+ \begin{equation*}
+ R_{U\! @AU^*}(\lambda) = U R_A(\lambda) @ U^*\!.
+ \end{equation*}
+ Since conjugation with an unitary commutes with the strong operator limit, we obtain
+ \begin{equation*}
+ E_{U\! @AU^*} \parens[\big]{\bracks{a,b}}
+ = U E_A \parens[\big]{\bracks{a,b}} U^*
+ \end{equation*}
+ for all $a,b \in \RR$.
+ The collection $\mathcal{A}$ of all subsets $S$ of $\RR$ such that
+ $E_{U\! @AU^*} \parens[\big]{S} = U E_A \parens[\big]{S} U^*$
+ is a $\sigma$-algebra on $\RR$.
+ We have shown that all closed intervals belong to $\mathcal{A}$.
+ It is well known that the Borel-$\sigma$-algebra $\mathcal{B}$ of $\RR$
+ is generated by the closed intervals. Hence, $\mathcal{B} \subset \mathcal{A}$.
+ This shows that the spectral measures $U E_A U^*$ and $E_{U\! @AU^*}$ coincide.
+\end{proof}
+
+\begin{equation*}
+ \Delta_W^{it}
+ = U(g) \Delta^{it} U(g)^*
+ = U(g) U\parens[\big]{0,\Lambda(t)} U(g)^*
+ = U\parens[\big]{g(0,\Lambda(t))g^{-1}}
+\end{equation*}
+
+
+Recall that Stones Theorem \todo{add reference} states that
+every strongly continuous one-parameter unitary group
+is of the form $t \mapsto e^{itK}$ with a uniquely determined
+selfadjoint operator $K$, which is called \emph{infinitesimal generator} of the group.
+
+\begin{definition}{Modular Hamiltonian}{}
+ The infinitesimal generator of the modular group associated to a spacetime region $\spacetimeregion{O}$ is called the
+ \emph{modular Hamiltonian}\index{modular!Hamiltonian}\nomenclature{$K_{\spacetimeregion{O}}$}{modular Hamiltonian for $\spacetimeregion{O}$}
+ for said region, and denoted $K_{\spacetimeregion{O}}$.
+\end{definition}
+
+In other words, $K_{\spacetimeregion{O}}$ is the unique selfadjoint operator such that $\Delta_{\spacetimeregion{O}}^{it} = e^{itK_{\spacetimeregion{O}}}$ for all $t \in \RR$.
+
+\begin{proposition}{}{}
+ The modular Hamiltonian for the right wedge is given by $d \Gamma(A)$, where
+ \begin{equation*}
+ A\psi(p) = - \frac{2\pi}{i} \parens[\big]{\partial_0 \psi(p) \, p^1 + \partial_1 \psi(p) \, p^0}
+ \end{equation*}
+\end{proposition}
+
+\section{Complex Lorentz Transformations}
+
+\subsection{Analytic Continuation of the Space-Time Translation Group}
+
+\subsection{Complex Lorentz Boosts}
+
+\begin{lemma}{}{}
+ Suppose $A$ is a selfadjoint operator on some Hilbert space $\hilb{H}$.
+ For all complex numbers $z$ define a closed normal operator $V(z) = e^{izA}$ by means of functional calculus.
+ Let $g$ be a xxx function. Then the range of the bounded operator $g(A)$ is contained in the domain of $V(z)$ for all $z$, and
+ \begin{equation*}
+ V(z) g(A) = \int e^{iz \lambda} g(\lambda) dE_A(\lambda).
+ \end{equation*}
+\end{lemma}
+
+\subsection{A Convolution Theorem for Vector-Valued Tempered Distributions}
+
+\blockcquote{Bisognano1975}{%
+ The extension to vector-valued tempered distributions is trivial.
+}
+
+
+
+\chapterbib
+\cleardoublepage
+
+% vim: syntax=mytex
diff --git a/preamble.tex b/preamble.tex
index 4a88ef7..35d94ad 100644
--- a/preamble.tex
+++ b/preamble.tex
@@ -15,7 +15,7 @@
%\usepackage{graphicx}
\usepackage{tcolorbox}
\usepackage[style=ext-alphabetic]{biblatex}
-\usepackage[intoc]{nomencl}
+\usepackage[intoc,refpage]{nomencl}
\usepackage{makeidx}
\usepackage{idxlayout}
\usepackage{hyperref}
@@ -82,6 +82,7 @@
\numberwithin{equation}{chapter}
\DeclareMathOperator{\supp}{supp}
\DeclareMathOperator{\dom}{dom}
+\DeclareMathOperator{\ran}{ran}
% extend amsmath's proof environment
\NewDocumentEnvironment{myproof}{Ob}{\IfNoValueTF{#1}{\begin{proof}}{\begin{proof}[\proofname\ of \Cref{#1}]}}{\end{proof}}
@@ -177,7 +178,8 @@
% ---------- nomencl
\makenomenclature
\renewcommand*{\nomname}{List of Symbols}
-\def\pagedeclaration#1{, \hyperlink{page.#1}{page\nobreakspace#1}}
+%\def\pagedeclaration#1{, \hyperlink{page.#1}{page\nobreakspace#1}}
+\def\pagedeclaration#1{, \hyperlink{page.#1}{#1}}
% ---------- makeidx
\makeindex
@@ -250,6 +252,8 @@
% Fourier transformation
% ----------------------
\newcommand*{\ft}[1]{\hat{#1}}
+\newcommand*{\wideft}[1]{\widehat{#1}}
+\newcommand*{\ift}[1]{\check{#1}}
\newcommand*{\FT}[1]{\mathcal{F}\parens*{#1}}
\newcommand*{\iFT}[1]{\mathcal{F}^{-1}\parens*{#1}}
@@ -269,6 +273,7 @@
\newcommand*{\schwartz}[1]{\mathcal{S}(#1)}
\newcommand*{\realschwartz}[1]{\mathcal{S}_{\RR}(#1)}
\newcommand*{\tempdistrib}[1]{\mathcal{S}'(#1)}
+\newcommand*{\tempdistribnoarg}{\mathcal{S}'}
% Fock spaces
% -----------
@@ -289,6 +294,8 @@
% ---------
\newcommand*{\QF}[1]{QF(#1)}
\newcommand{\QFequal}{\overset{\text{\scriptsize QF}}{=}}
+% operator associated to a quadratic form
+\newcommand*{\QFop}[1]{{#1}_{\mathrm{op}}}
% Standard Subspaces
% ------------------
@@ -311,3 +318,29 @@
}}
\newcommand{\defequal}{\overset{\text{\scriptsize def}}{=}}
+
+\newcommand*{\energydensity}{\varrho}
+\newcommand*{\fockvaccum}{\Omega}
+
+% Observable Algebras
+\newcommand*{\vNa}[1]{\mathcal{#1}}
+\newcommand*{\localalg}[1]{\vNa{R}(#1)}
+
+
+\newcommand*{\FullLorentzGroup}{\mathcal{L}}
+\newcommand*{\ProperOrthochronousLorentzGroup}{\FullLorentzGroup_{+}^{\uparrow}}
+\newcommand*{\FullPoincareGroup}{\mathcal{P}}
+\newcommand*{\ProperOrthochronousPoincareGroup}{\FullPoincareGroup_{+}^{\uparrow}}
+
+% spacetime domains
+\newcommand*{\spacetimeregion}[1]{\mathcal{#1}}
+\newcommand*{\rightwedge}{W_{\! R}}
+\newcommand*{\leftwedge}{W_{\! L}}
+
+\newcommand*{\todo}[1]{{\color{blue}TODO: #1}}
+\newcommand*{\question}[1]{{\color{blue}Question: #1}}
+\newcommand*{\info}[1]{{\color{blue}Info: #1}}
+
+\newcommand*{\operatorclosure}[1]{\overline{#1}}
+
+\DeclareMathOperator*{\stronglim}{s-lim}
diff --git a/stresstensor.tex b/stresstensor.tex
index a4bb6fb..4c128c2 100644
--- a/stresstensor.tex
+++ b/stresstensor.tex
@@ -35,9 +35,13 @@ as a service to the reader.
\item Given a complex-valued function $f$ on $M$, we define its \emph{Fourier transform} $\ft{f}\,$ by
\begin{equation}
\label{fourier-transform}
- \hat{f}(p) = \frac{1}{(2 \pi)^2} \int_{M} e^{i p \cdot x} f(x) \, dx
+ \ft{f}(p) \defequal \int_{M} e^{i p \cdot x} f(x) \, dx
\end{equation}
- whenever the integral converges. The \emph{inverse Fourier transform} is TODO
+ whenever the integral converges. The \emph{inverse Fourier transform} is defined by
+ \begin{equation*}
+ \label{inverse-fourier-transform}
+ \ift{f}(p) \defequal \frac{1}{(2 \pi)^2} \int_{M} e^{-i p \cdot x} f(x) \, dx.
+ \end{equation*}
\item To a mathematician $\overline{\phantom{z}}$ usually means complex conjugation and ${}^*$ indicates the Hilbert adjoint of an operator,
while a physicist may read ${}^*$ as complex conjugation and
denotes the Hilbert adjoint with ${}^{\dagger}$.
@@ -85,9 +89,15 @@ as a service to the reader.
\begin{equation*}
E : \schwartz{M} \to \hilb{H}, \quad f \mapsto Ef = \left.\ft{f}\,\right\vert {X_m^+}
\end{equation*}
+ We define a $\RR$-linear mapping $\phi$ by
+ \begin{equation*}
+ \realschwartz{M} \ni f \mapsto \varphi(f) = \Phi_{\mathrm{S}}(Ef) = \frac{1}{\sqrt{2}} \parens*{a(Ef) + a(Ef)^\dagger}
+ \end{equation*}
+ This extedns to complex valued test functions $f \in \schwartz{M}$
\begin{equation*}
- \realschwartz{M} \ni f \mapsto \Phi(f) = \Phi_{\mathrm{S}}(Ef) = \frac{1}{\sqrt{2}} \parens*{a(Ef) + a(Ef)^\dagger}
+ \varphi(f) = \frac{1}{\sqrt{2}} \parens*{a(E\bar{f}) + a(Ef)^\dagger}
\end{equation*}
+ called the \emph{massive free scalar quantum field}
\item
annihilation and creation operators, $f \in \schwartz{M}$, $\psi \in \BosonFock{\hilb{H}}$
\begin{align*}
@@ -176,7 +186,7 @@ leads to
\parens[\big]{a(p)^\dagger \psi} {}_n (k_1, \ldots, k_n)
= \frac{1}{\sqrt{n}} \sum_{i=1}^n \delta(p - k_i) \, \psi_{n-1} (k_1, \ldots, \widehat{k_i}, \ldots, k_n),
\end{equation}
-where the symmetization is necessary to obtain an expression that
+where the symmetrization is necessary to obtain an expression that
at least has a chance of being a $n$ Boson state.
However, it clearly is not a $L^2$ function.
Given any state $\psi'$, we can
@@ -207,7 +217,7 @@ For completeness, we give a precise definition of quadratic form.
q : D(q) \times D(q) \to \CC,
\end{equation*}
where $D(q)$ is a linear subspace of $\hilb{H}$, called the \emph{form domain}\index{form domain}\index{quadratic form!domain of a},
- such that $q$ is conjugate linear in its first agrument
+ such that $q$ is conjugate linear in its first argument
and linear in its second argument (i.e.\ sesquilinear).
We say that $q$ is \emph{densely defined}
if $D(q)$ is dense in $\hilb{H}$.
@@ -221,13 +231,13 @@ but one may obtain an operator with trivial domain.
\begin{definition}{Operator Associated to a Quadratic Form}{}
Suppose $q$ is a densely defined quadratic form on a complex Hilbert space $\hilb{H}$.
- The linear \emph{operator associated to}\index{quadratic form!operator associated to a} $q$, denoted $q_{\mathrm{op}}$,
+ The linear \emph{operator associated to}\index{quadratic form!operator associated to a} $q$, denoted $\QFop{q}$,
is defined on the domain
\begin{equation*}
- D(q_{\mathrm{op}}) = \braces{\psi \in D(q) \mid \text{the map $q(\cdot,\psi) : D(q) \to \CC$ is bounded}},
+ D(\QFop{q}) = \braces{\psi \in D(q) \mid \text{the map $q(\cdot,\psi) \vcentcolon D(q) \to \CC$ is bounded}},
\end{equation*}
- and maps $\psi \in D(q_{\mathrm{op}})$ to the vector $q_{\mathrm{op}}\psi$ in $\hilb{H}$ satisfying
- $q(\psi',\psi) = \innerp{\psi'}{q_{\mathrm{op}}\psi}$,
+ and maps $\psi \in D(\QFop{q})$ to the vector $\QFop{q}\psi$ in $\hilb{H}$ satisfying
+ $q(\psi'\!,\psi) = \innerp{\psi'\!}{\QFop{q}\psi}$,
which exists and is unique by Riesz’s Representation Theorem.
\end{definition}
@@ -299,7 +309,7 @@ the $\alpha^{(0)},\alpha^{(1)}_i,\alpha^{(2)}_{j,k},\ldots$ are complex numbers,
of which only finitely many are nonzero,
and $e$ is a special object representing an empty product of $z$'s.
To make this mathematically precise:
-we are speaking of the noncommutative associative algebra over $\CC$
+we are speaking of the non-commutative associative algebra over $\CC$
freely generated by the elements of $\hilb{H}$.
The unit of the algebra is $e$.
@@ -314,7 +324,7 @@ where $z,z' \in \hilb{H}$.
\begin{definition}{Infinitesimal Weyl Algebra}{}
Let $\hilb{H}$ be a complex Hilbert space.
The \emph{infinitesimal Weyl algebra}\index{infinitesimal Weyl algebra} $\WeylAlg(\hilb{H})$ over $\hilb{H}$
- is the noncommutative associative algebra over $\CC$
+ is the non-commutative associative algebra over $\CC$
generated by the elements of $\hilb{H}$, with the relations
\begin{equation*}
zz' - z'z = i \Imag \innerp{z}{z'} \, e \qquad z,z' \in \hilb{H},
@@ -371,10 +381,10 @@ the formula makes sense even for $r=0$ and asserts that $\normord{e} = e$.
The cases $r=1$ and $r=2$ read
\begin{align*}
\normord{z} &=
- \frac{1}{\sqrt{2}} \parens[\big]{\weylannihilator(z) + \weylcreator(z)} = z \\
+ \frac{1}{\sqrt{2}} \parens[\big]{\weylannihilator(z) + \weylcreator(z)} = z, \\
\normord{z_1 z_2} &= \frac{1}{2}
- \parens[\big]{\weylannihilator(z_1) \weylannihilator(z_2) + \weylannihilator(z_1) \weylcreator(z_2)
- + \weylcreator(z_1) \weylannihilator(z_2) + \weylcreator(z_1) \weylcreator(z_2) }
+ \parens[\big]{ \weylannihilator(z_1) \weylannihilator(z_2) + \weylcreator(z_1) \weylannihilator(z_2)
+ + \weylcreator(z_2) \weylannihilator(z_1) + \weylcreator(z_1) \weylcreator(z_2) }.
\end{align*}
This suggests that the normally ordered product $\normord{z_1 \!\cdots z_r}$
is symmetric in $z_1,\ldots,z_n$. This is in fact true, and becomes evident
@@ -470,20 +480,19 @@ and may be obtained via integration by parts.
Naturally, we now define the \emph{distributional derivative} of the field by
\begin{equation*}
- D \varphi(f) = \varphi(D^{\dagger} f) \qquad \forall f \in \schwartz{\RR^d}
+ D \varphi(f) = \varphi(D^{\dagger} f) \qquad \forall f \in \schwartz{\RR^4}
\end{equation*}
-As one expects, $D\varphi$ is an operator-valued tempered distribution on $M=\RR^d$.
-TODO
+As one expects, $D\varphi$ is an operator-valued tempered distribution on $M=\RR^4$.
+In terms of creation and annihilation operators we have
\begin{equation}
\label{derivative-free-field}
- D \varphi(f) = \frac{1}{\sqrt{2}} \parens*{a(ED^{\dagger}f)^{\dagger} + a(ED^{\dagger}f)}
+ D \varphi(f) = \frac{1}{\sqrt{2}} \parens*{a(ED^{\dagger}f)^{\dagger} + a(E\overline{D^{\dagger}f})}.
\end{equation}
-
-
-The operator corresponding to $D$ in Fourier space is the multiplication operator
+In Fourier space the operator $D^\dagger$ corresponds to muliplication with the polynomial
\begin{equation*}
- -i \sum_{\alpha} a_{\alpha} p_0^{\alpha_0} (-p_1)^{\alpha_1} (-p_2)^{\alpha_2} (-p_3)^{\alpha_3}
+ \ft{D}(p) \defequal \sum_{\alpha} i^{\abs{\alpha}} a_{\alpha} (+p^0)^{\alpha_0} (-p^1)^{\alpha_1} (-p^2)^{\alpha_2} (-p^3)^{\alpha_3}
\end{equation*}
+If $D=\partial^{\mu}$, then $\ft{D}(p) = i @ p_{\!\mu}$, were the potential sign is concealed by lowering the index.
Let $D_1, \ldots, D_r$ be linear differential operators with constant (complex) coefficients.
@@ -494,8 +503,8 @@ Let $D_1, \ldots, D_r$ be linear differential operators with constant (complex)
\sum_{\sigma \in S_r}
\sum_{s=0\vphantom{S}}^{r}
\frac{1}{s!(r-s)!}
- \prod_{i=1\vphantom{S}}^{s} a^\dagger(D^\dagger_{\sigma(i)}f)
- \prod_{\mathclap{j=s+1\vphantom{S}}}^{r} a(D^\dagger_{\sigma(j)}f)
+ \prod_{i=1\vphantom{S}}^{s} a^\dagger(ED^\dagger_{\sigma(i)}f)
+ \prod_{\mathclap{j=s+1\vphantom{S}}}^{r} a(E\overline{D^\dagger_{\sigma(j)}f})
\end{gather}
\section{Renormalized Products of the Free Field and~its~Derivatives}
@@ -516,7 +525,7 @@ this approach incurs significant technical difficulties.
\begin{lemma}{Integral Representation of the Renormalized Product}{renormalized-product-integral-representation}
Let $\varphi$ be the free scalar quantum field with mass parameter $m > 0$.
Let $D_1, \ldots, D_r$ be linear differential operators with constant (complex) coefficients.
- Then, for arbitrary Schwartz functions $f \in \schwartz{M}$ and Fock states $\psi,\psi' \in \BosonFock{L^2(X_m^+,d\Omega_m)}$,
+ Then, for arbitrary Schwartz functions $f \in \realschwartz{M}$ and Fock states $\psi,\psi' \in \BosonFock{L^2(X_m^+,d\Omega_m)}$,
we have
\begin{equation*}
\innerp{\psi'\!}{\normord{D_1 \varphi(f) \cdots D_r \varphi(f)} \,\psi} =
@@ -552,7 +561,32 @@ this approach incurs significant technical difficulties.
\end{multline*}
\end{lemma}
-TODO(Note about the remaining dependence of $K$ on $f$.)
+Note that $K$ has a remaining dependence on $f$ via $\chi$
+even thogh the notation does not indicate this.
+This is made explicit in the alternative integral representation
+ \begin{equation}
+ \label{equation:alternative-integral-representation}
+ \begin{multlined}
+ \innerp{\psi'\!}{\normord{D_1 \varphi(f) \cdots D_r \varphi(f)} \,\psi} =\\
+ \hspace{1cm} \int dp_1 \!\cdots dp_r
+ \sum_{s=0}^{r}
+ \, \ft{f}(p_1) \cdots\! \ft{f}(p_s)
+ \, \overline{\ft{f}(p_{s+1}) \cdots\! \ft{f}(p_r)}
+ \, \tilde{K}^s_{\psi'\!,\psi}(p_1,\ldots,p_r)
+ \end{multlined}
+ \end{equation}
+ where
+ \begin{multline*}
+ \tilde{K}^s_{\psi'\!,\psi}(p_1,\ldots,p_r) =
+ P_s(p_1,\ldots,p_r)
+ \sum_{m=0}^{\infty} \sum_{n=0}^{\infty}
+ \delta_{m-s}^{n-(r-s)} \\
+ \cdot \sqrt{m(m-1) \cdots (m-s+1)} \sqrt{n(n-1) \cdots (n-(r-s)+1)} \\
+ \cdot \int dk_1 \cdots dk_{m-s}
+ \ \overline{\psi'_m(k_1,\ldots,k_{m-s},p_1,\ldots,p_s)}
+ \ \psi_n(k_1,\ldots,k_{n-(r-s)},p_{s+1},\ldots,p_r).
+ \end{multline*}
+ This will be more convenient for xxx
\begin{myproof}[lemma:renormalized-product-integral-representation]
From equation~\eqref{equation:renormalized-product},
@@ -617,24 +651,30 @@ In particular, for squares ($r=2$) we have
The following assertion is key to realizing the idea of taking the limit $f \to \delta_x$.
-\begin{lemma}{}{integral-kernel-h-bound}
+\begin{lemma}{H-bounds for the Integral Kernel}{integral-kernel-h-bound}
In the setting of \cref{lemma:renormalized-product-integral-representation},
there exist a constant $C$, and a positive integer $l$,
- such that for arbitrary states $\psi,\psi' \in \BosonFock{\hilb{H}}$,
- and test functions $f \in \schwartz{M}$,
+ such that for arbitrary test functions $f \in \schwartz{M}$
+ and states $\psi,\psi' \in \Domain{H^l}$,
the function $K_{\psi'\!,\psi}$ is integrable (that is, $L^1$)
and satisfies the $H$-bound
\begin{equation*}
\norm{K_{\psi'\!,\psi}}_1 \le
C \norm{(1+H)^l \psi'} \norm{(1+H)^l \psi}.
\end{equation*}
+ More specifically, it is sufficient to choose $l > rd + r/2$,
+ where $d$ is the highest order of differentiation occuring in $D_1, \ldots, D_r$.
\end{lemma}
-The Hamilton operator $H$ acts on $n$-particle states $\psi_n$
-by multiplication with $\omega(p_1)$
-In the following proof it will we convenient to use the abbreviation
+The Hamilton operator $H$ acts on $n$-particle states $\psi_n$ as follows:
+\begin{gather*}
+ H \psi_n(p_1,\ldots,p_n) = \parens[\big]{\omega(p_1) + \cdots + \omega(p_n)} \psi_n(p_1,\ldots,p_n) \\
+ \shortintertext{where}
+ \omega(p) = \omega(\symbfit{p}) = \sqrt{m^2 + \abs{\symbfit{p}}^2} = \sqrt{m^2 + (p^1)^2 + (p^2)^2 + (p^3)^2}.
+\end{gather*}
+In the following proof it will be convenient to use the abbreviation
\begin{equation*}
- \omega(p_1,\ldots,p_s) = \omega(p_1) + \cdots + \omega(p_s)
+ \omega(p_1,\ldots,p_s) \defequal \omega(p_1) + \cdots + \omega(p_n).
\end{equation*}
\begin{myproof}[lemma:integral-kernel-h-bound]
@@ -778,28 +818,71 @@ In the following proof it will we convenient to use the abbreviation
\le \frac{\omega(p_1) + \cdots + \omega(p_s)}{s}
\le \omega(p') \le 1 + \omega(k,p'), \nonumber\\
\shortintertext{hence}
- \label{equation:one-plus-omega-estimate}
+ \label{equation:one-plus-omega-estimate1}
\parens[\big]{1+\omega(k,p')} {}^{-a}
- \le \parens[\big]{\omega(p_1) \cdots \omega(p_s)} {}^{-a/s}.
+ \le \parens[\big]{\omega(p_1) \cdots \omega(p_s)} {}^{-a/s}, \\
+ \shortintertext{and similarly}
+ \label{equation:one-plus-omega-estimate2}
+ \parens[\big]{1+\omega(k,p)} {}^{-a}
+ \le \parens[\big]{\omega(p_1) \cdots \omega(p_{r-s})} {}^{-a/(r-s)}.
\end{gather}
- The estimates~\eqref{equation:polynomial-estimate} and~\eqref{equation:one-plus-omega-estimate} entail
+ The estimates~\eqref{equation:polynomial-estimate},~\eqref{equation:one-plus-omega-estimate1} and~\eqref{equation:one-plus-omega-estimate2} entail
\begin{equation*}
\abs{F(k,p',p)} \le C_s
\prod_{i=1}^{s} \omega(p_i)^{d_i-a/s}
- \prod_{j=s+1}^{r} \omega(p_j)^{d_j-a/(r-s)}
+ \prod_{j=s+1}^{r} \omega(p_j)^{d_j-a/(r-s)}.
\end{equation*}
+ Since the right hand side does not depend on $k$, and the $p$-variables are separated,
+ the problem reduces to proving that
+ \begin{equation}
+ \label{equation:integral-finite}
+ \int \omega(q)^{-2b} \,d\Omega(q) < \infty
+ \end{equation}
+ for $b$ large enough.
+ Recall that $d \Omega(q) = \omega(q)^{-1} d^3 \symbfit{q}$.
+ By transformation to spherical coordinates, we find that~\eqref{equation:integral-finite}
+ is equivalent to
+ \begin{equation*}
+ \int \frac{r^2}{(m^2 + r^2)^{b+1/2}} \,dr < \infty
+ \end{equation*}
+ It it well known that this holds for $b > 1$.
+ $a > r d$
+
+ \begin{equation}
+ \label{equation:intermediate-result}
+ \norm{K_{\psi'\!,\psi}}_1 \le
+ \sum_{m=0}^{\infty} \sum_{n=0}^{\infty}
+ \sum_{s=0}^{r} \delta_{m-s}^{n-(r-s)} C_s
+ \underbracket{\norm{(1+H)^l \psi'_m}_2}_{a'_m}
+ \underbracket{\norm{(1+H)^l \psi_n}_2}_{a_n}
+ \end{equation}
+ We introduce auxiliary variables $a'_m, a_n$ as shown above, and for convenience set $a_n = 0$ whenever $n < 0$.
+ Using this, we rewrite the right hand side of~\eqref{equation:intermediate-result}
+ and apply the Cauchy-Schwarz Inequality for sequences as follows:
+ \begin{equation*}
+ \sum_{s=0}^{r} C_s \sum_{m = 0}^{\infty} a'_m \cdot a_{m+r-2s} \le
+ \sum_{s=0}^{r} C_s \sqrt{\sum_{m=0}^{\infty} a'^2_m \sum_{n=0}^{\infty} a^2_n}
+ \end{equation*}
+ To complete the proof, observe that
+ \begin{equation*}
+ \norm{(1+H)^l \psi'}_2 =
+ \sqrt{\sum_{m=0}^{\infty} \norm{(1+H)^l \psi'_m}_2^2} =
+ \sqrt{\sum_{m=0}^{\infty} a'^2_m},
+ \end{equation*}
+ and similar for $\psi$, by definition of the inner prouct
+ and because $((1+H)^l \psi')_m = (1+H)^l \psi'_m$ for all $m$.
\end{myproof}
\begin{lemma}{Renormalized Product at a Point}{}
In the setting of \cref{lemma:renormalized-product-integral-representation},
- assume that $\psi,\psi'$ are in $D^l(H)$.
+ assume that $\psi,\psi'$ are in $\Domain{H^l}$.
Let $x$ be any point in $M$ and let $\delta_x \in \tempdistrib{M}$ be the Dirac distribution supported in $x$.
Then the limit
\begin{equation*}
\lim_{f \to \delta_x}
\innerp{\psi'\!}{\normord{D_1 \varphi(f) \cdots D_r \varphi(f)} \,\psi}
\end{equation*}
- exists and depends continously on $x$.
+ exists and depends continuously on $x$.
\end{lemma}
\begin{proof}
@@ -813,8 +896,22 @@ In the following proof it will we convenient to use the abbreviation
The integrand is dominated by the function $\abs{K_{\psi'\!,\psi}(p_1,\ldots,p_r)}$,
which has finite integral as it is $L^1$
by \cref{lemma:integral-kernel-h-bound}.
+
Moreover, the integrand converges pointwise to $K_{\psi'\!,\psi}(p_1,\ldots,p_r)$, since $\ft{f} \to 1$ when $f \to \delta_x$.
TODO(With of choice of FT constants, $\ft{f} \to 1/(2\pi)^2$. Change here or change def?)
+
+ Since the Fourier transformation of tempered distribution
+ is a continuous mapping $\tempdistribnoarg \to \tempdistribnoarg$,
+ we have $\ft{f} \to \FT{\delta_x}$ whenever $f \to \delta_x$ in the topology of $\tempdistribnoarg$.
+ Recall that $\ft{\delta} = 1$, and thus $\FT{\delta_x}(p) = e^{ix \cdot p}$ for all $p \in M$.
+ This shows that the integrand converges pointwise to
+ \begin{equation*}
+ \sum_{s=0}^r
+ e^{ix \cdot (p_1 + \cdots + p_s)}
+ e^{-ix \cdot (p_{s+1} + \cdots + p_r)}
+ \tilde{K}_{\psi'\!,\psi}(p_1,\ldots,p_r)
+ \end{equation*}
+
The Dominated Convergence Theorem implies
\end{proof}
@@ -847,20 +944,21 @@ In the following proof it will we convenient to use the abbreviation
\begin{lemma}{TODO}{}
Let $\varphi$ be a free quantum field.
Let $D_1, \ldots, D_r$ be linear differential operators with constant (complex) coefficients.
- Then we have for all states $\psi,\psi' \in D^l(H)$
+ Suppose that $l$ is a positive integer large enough to satisfy the
+ Then we have for all states $\psi,\psi' \in \Domain{H^l}$
\begin{multline*}
\innerp{\psi'\!}{\normord{D_1 \varphi \cdots D_r \varphi}(f) \,\psi} = \\
= \int dp_1 \!\cdots dp_r
\sum_{s=0}^{r}
\, \ft{f}(p_1 + \cdots + p_s - p_{s+1} - \cdots - p_r)
- \, L_{\psi'\!,\psi}^{s}(p_1,\ldots,p_r)
+ \, \tilde{K}_{\psi'\!,\psi}^{s}(p_1,\ldots,p_r)
\end{multline*}
where
\begin{multline*}
- L_{\psi'\!,\psi}^{s}(p_1,\ldots,p_r) =
+ \tilde{K}_{\psi'\!,\psi}^{s}(p_1,\ldots,p_r) =
+ P_s(p_1,\ldots,p_r)
\sum_{m=0}^{\infty} \sum_{n=0}^{\infty}
- \delta_{m-s}^{n-(r-s)}
- \ P_s(p_1,\ldots,p_r) \\
+ \delta_{m-s}^{n-(r-s)} \\
\cdot \sqrt{m(m-1) \cdots (m-s+1)} \sqrt{n(n-1) \cdots (n-(r-s)+1)} \\
\cdot \int dk_1 \cdots dk_{m-s}
\ \overline{\psi'_m(k_1,\ldots,k_{m-s},p_1,\ldots,p_s)}
@@ -869,10 +967,12 @@ In the following proof it will we convenient to use the abbreviation
and $P_s(p_1,\ldots,p_r)$ is defined as before.
\end{lemma}
-%\[
- %f(T), f\left( T \right),
- %\int_{a}^{b} f\left( x \right) d x, \frac{1}{T},
-%\]
+\begin{proof}
+ a
+\end{proof}
+
+
+\section{Definition of the Stress Tensor}
In the theory of a real scalar field $\phi$ of mass $m$,
the Lagrangian density of the Klein-Gordon action is given by
@@ -890,7 +990,7 @@ Raising the index $\nu$ and inserting \cref{lagrangian-density} yields
\end{equation*}
The \emph{energy density}:
\begin{equation*}
- \rho = T^{00} = \frac{1}{2} \parens*{\sum_{\mu=0}^{3} (\partial^{\mu}\phi)^2 + m^2 \phi^2}
+ \energydensity = T^{00} = \frac{1}{2} \parens*{\sum_{\mu=0}^{3} (\partial^{\mu}\phi)^2 + m^2 \phi^2}
\end{equation*}
The discussion in the previous section enables us to define
the \emph{renormalized stress-energy tensor} of a free scalar field $\varphi$ by
@@ -900,11 +1000,11 @@ the \emph{renormalized stress-energy tensor} of a free scalar field $\varphi$ by
and this is a quadratic form.
In particular, the energy density is
\begin{equation*}
- \rho = \frac{1}{2} \sum_{\mu=0}^{3} \normord{(\partial^{\mu}\phi)^2} + \frac{1}{2} m^2 \normord{\phi^2}
+ \energydensity = \frac{1}{2} \sum_{\mu=0}^{3} \normord{(\partial^{\mu}\varphi)^2} + \frac{1}{2} m^2 \normord{\varphi^2}
\end{equation*}
\begin{multline*}
- \innerp{\psi'\!}{\rho(f) \,\psi} = \\
+ \innerp{\psi'\!}{\energydensity(f) \,\psi} = \\
= \int dp_1 dp_2
\parens{p_1^{\mu} p_2^{\mu} + m^2}
\sum_{s=0}^{r} (-1)^{s+1}
@@ -925,13 +1025,13 @@ where
\begin{theorem}{TODO}{}
Let $\varphi$ be a free quantum field.
Let $D_1, \ldots, D_r$ be linear differential operators with constant (complex) coefficients.
- Then we have for all states $\psi,\psi' \in D^l(H)$
+ Then we have for all test functions $f \in \schwartz{M}$
\begin{multline*}
\normord{D_1 \varphi \cdots D_r \varphi}(f) \QFequal \int dp_1 \!\cdots dp_r \sum_{s=0}^{r}
P_s(p_1,\ldots,p_r) \, \ft{f}(p_1 + \cdots + p_s - p_{s+1} - \cdots - p_r) \\
\cdot a^\dagger(p_1) \cdots a^\dagger(p_s) a(p_{s+1}) \cdots a(p_r)
\end{multline*}
- as quadratic forms on $D^l(H)$, where
+ as quadratic forms on $\Domain{H^l}$, where
\begin{multline*}
\quad P_s(p_1,\ldots,p_r) =
\frac{1}{\sqrt{2^r}}
@@ -944,17 +1044,107 @@ where
\begin{definition}{}{}
\begin{multline*}
- \rho(f) \QFequal \frac{1}{4} \int dp dp' (p \cdot p' + m^2)
+ \energydensity(f) \QFequal \frac{1}{4} \int dp dp' (p \cdot p' + m^2)
\Big\lbrack \ft{f}(p+p') a(p) a(p') + {}\\
+ 2\ft{f}(p-p') a^\dagger(p) a(p') + \ft{f}(-p-p') a^\dagger(p) a^\dagger(p') \Big\rbrack
\end{multline*}
\end{definition}
+\begin{equation*}
+ \bar{p} := \eta p = (p^0,-\symbfit{p})
+\end{equation*}
+
+\begin{proposition}{}{}
+ \begin{multline*}
+ \innerp{\psi'}{\energydensity(f) \psi} =
+ \frac{1}{4} \int dp dp'
+ (\bar{p} \cdot p' + m^2)
+ \bracks[\big]{2 \ft{f}(p - p') L^1_{\psi'\!,\psi}(p,p')} \\
+ + (-\bar{p} \cdot p' + m^2)
+ \bracks[\big]{\ft{f}(- p - p') L^0_{\psi'\!,\psi}(p,p') + \ft{f}(p + p') L^2_{\psi'\!,\psi}(p,p')}
+ \end{multline*}
+\end{proposition}
+
+\begin{proposition}{}{}
+ \begin{multline*}
+ \energydensity(f) \QFequal \frac{1}{4} \int dp dp'
+ (m^2 + \bar{p} \cdot p')
+ \bracks[\Big]{2\ft{f}(p-p') a^\dagger(p) a(p')} \\
+ + (m^2 - \bar{p} \cdot p')
+ \bracks[\Big]{\ft{f}(p+p') a(p) a(p') + \ft{f}(-p-p') a^\dagger(p) a^\dagger(p')}
+ \end{multline*}
+\end{proposition}
+
+\begin{proposition}{}{}
+ The Fock vaccum $\fockvaccum$ lies in the domain of $\energydensity(f)\QFop{}$
+ for all test functions $f \in \schwartz{M}$
+ and $\energydensity(f)\QFop{}\fockvaccum$ is the vector $\psi$ defined by
+ \begin{equation*}
+ \psi_2(p,p') = \frac{\sqrt{2}}{4} (m^2 - \bar{p} \cdot p') \ft{f}(-p-p')
+ \end{equation*}
+ and $\psi_n \equiv 0$ for $n \ne 2$.
+\end{proposition}
+
+\begin{equation*}
+ \energydensity(f) \Omega = ?
+\end{equation*}
+
\section{Essential Selfadjointness of Renormalized Products}
-a
+\begin{lemma}{H-Bounds for the Renormalized Product}{}
+ \begin{equation*}
+ \abs{\innerp{\psi'\!}{\normord{D_1 \varphi \cdots D_r \varphi}(f)\psi}} \le
+ C \norm{(I+H)^l \psi} \norm{(I+H)^l \psi'}
+ \end{equation*}
+\end{lemma}
-%\nocite{*}
+\begin{proof}
+ This proof is nearly identical to that of \cref{lemma:integral-kernel-h-bound}
+ and we will only cover the differences.
+ \begin{equation*}
+ \begin{multlined}[c]
+ \abs{\innerp{\psi'\!}{\normord{D_1 \varphi \cdots D_r \varphi}(f)\psi}} \le
+ \sum_{m=0}^{\infty} \sum_{n=0}^{\infty} \sum_{s=0}^{r}
+ \delta_{m-s}^{n-(r-s)} \\
+ \hspace{2.5cm} \cdot
+ \int \!dk \int \!dp'\! \int \!dp \, \abs*{F(k,p',p) \, G'(k,p') \, G(k,p)},
+ \end{multlined}
+ \end{equation*}
+ where
+ \begin{multline}
+ F(k,p',p) = \parens[\big]{1+\omega(k,p')} {}^{-a} \parens[\big]{1+\omega(k,p)} {}^{-a} P_s(p',p) \\
+ \cdot \ft{f}(p_1 + \cdots + p_s - p_{s+1} - \cdots - p_r)
+ \end{multline}
+ and $G$ and $G'$ are defined as before.
+ All we have to do, is verifying that
+ \begin{equation*}
+ \sup_k \norm{F(k,\cdot,\cdot)}_2 < \infty
+ \end{equation*}
+ for a sufficiently large integer $a$.
+ Then we obtain the desired $H$-bound with $l=a+r/2$.
+
+ Recall that the Schwartz class is preserved by Fourier transform, translation and multiplication with polynomials.
+ Moreover, it is well known that Schwartz functions are square-integrable with repect to the Lorentz invariant measure on the mass shell.
+ Hence,
+ \begin{equation*}
+ \int dp_1 \abs{\ft{f}(p_1 + \cdots + p_s - p_{s+1} - \cdots - p_r)}^2
+ \end{equation*}
+ is bounded by a constant independent of $p_2,\ldots,p_r$.
+\end{proof}
+
+\section{Covariance}
+
+\begin{equation*}
+ f_g(x) \defequal f(g^{-1} x) \qquad
+ x \in M, \quad g \in \ProperOrthochronousPoincareGroup.
+\end{equation*}
+
+\begin{theorem}{Covariance}{covariance-renormalized-product}
+ \begin{equation*}
+ U(g) \,\normord{D_1 \varphi \cdots D_r \varphi}(f)\, U(g)^{-1}
+ = \normord{D_1 \varphi \cdots D_r \varphi}(f_g)
+ \end{equation*}
+\end{theorem}
\chapterbib
\cleardoublepage